Đến nội dung

anhquannbk nội dung

Có 474 mục bởi anhquannbk (Tìm giới hạn từ 24-05-2020)



Sắp theo                Sắp xếp  

#616450 xác định điểm M trong không gian

Đã gửi bởi anhquannbk on 22-02-2016 - 20:53 trong Hình học không gian

cho tứ diện ABCD.xác định điểm M trong không gian sao cho $MA^{2}+MB^{2}+MC^{2}+MD^{2}$ min

Gọi $ G $ là trọng tâm tứ diện

Ta có $ \overrightarrow{GA}+\overrightarrow{GB}+\overrightarrow{GC}+\overrightarrow{GD}=0 $

$ MA^{2}+ MB^{2} +MC^{2} +MD^{2} =(\overrightarrow{GA}-\overrightarrow{GM})^{2}+(\overrightarrow{GB}-\overrightarrow{GM})^{2}+(\overrightarrow{GC}-\overrightarrow{GM})^{2}+(\overrightarrow{GD}-\overrightarrow{GM})^{2}= GA^{2}+GB^{2}+GC^{2}+GD^{2}+4GM^{2}-2\overrightarrow{GM}( \overrightarrow{GA}+\overrightarrow{GB}+\overrightarrow{GC}+\overrightarrow{GD}) =+4GM^{2} $

 MA^{2}+ MB^{2} +MC^{2} +MD^{2}  $ min khi $ G $ trùng $ M $ vì $  MA^{2}+ MB^{2} +MC^{2} +MD^{2} $ không đổi




#605414 Xác định vị trí của A để AB.AC lớn nhất.

Đã gửi bởi anhquannbk on 26-12-2015 - 21:20 trong Hình học

1)Điểm A di động trên cung lớn BC nên $ \angle BAC $ nhọn

Ta $ S_{ABC}=\dfrac{1}{2}.AB.AC.sin\angle BAC $

Do $sin\angle BAC $ không đổi AB.AC lớn nhất khi $ S_{ABC} $ lớn nhất $ S_{ABC} $ lớn nhất khi AH lớn nhất(H hình chiếu của A trên BC)

hay A điểm chính giữa cung lớn BC




#605415 Xác định vị trí của A để AB.AC lớn nhất.

Đã gửi bởi anhquannbk on 26-12-2015 - 21:21 trong Hình học

1) Cho A là điểm di động trên cung lớn BC của (O;R). Xác định vị trí của A để AB.AC lớn nhất.

2) A là điểm cố định trên (O;R). AB, AC là 2 dây cung di động và AB.AC không đổi. Chứng minh BC luôn tiếp xúc với đường tròn cố định

1)Điểm A di động trên cung lớn BC nên $ \angle BAC $ nhọn

Ta  $ S_{ABC}=\dfrac{1}{2}.AB.AC.sin\angle BAC $

Do $sin\angle BAC $ không đổi AB.AC lớn nhất khi $ S_{ABC} $ lớn nhất $ S_{ABC} $ lớn nhất khi AH lớn nhất(H  hình chiếu của A trên BC)

hay A  điểm chính giữa cung lớn BC




#604862 Xác suất

Đã gửi bởi anhquannbk on 23-12-2015 - 16:54 trong Tổ hợp - Xác suất và thống kê - Số phức

Năm khách hàng vào một siêu thị nhỏ gồm 5 gian hàng khác nhau. Mỗi người chọn ngẫu nhiên một gian để mua hàng. Tính xác suất để mỗi gian hàng có đúng 1 người để đến mua




#644029 x,y,z>0,(x+y)(y+z)(z+x)=1.Cmr:

Đã gửi bởi anhquannbk on 07-07-2016 - 20:45 trong Bất đẳng thức và cực trị

em không biết .bài này thằng bạn đưa em làm, làm không ra nên post lên đây nhờ giải hộ.

em xem phần giải bài kỳ trước của báo tháng 5 sẽ có lời giải bài này.




#644026 x,y,z>0,(x+y)(y+z)(z+x)=1.Cmr:

Đã gửi bởi anhquannbk on 07-07-2016 - 20:41 trong Bất đẳng thức và cực trị

x,y,z>0,(x+y)(y+z)(x+z)=1.Cmr:

$\frac{\sqrt{x^{2}+xy+y^{2}}}{\sqrt{xy}+1}+\frac{\sqrt{y^{2}+yz+z^{2}}}{\sqrt{yz}+1}+\frac{\sqrt{z^{2}+zx+x^{2}}}{\sqrt{zx}+1}$ lớn hơn hay bằng căn 3

Bài này hình như đề ra kỳ này báo THTT tháng 1.




#639711 VMF's Marathon Đa thức Olympic

Đã gửi bởi anhquannbk on 12-06-2016 - 07:09 trong Đa thức

Lời giải bài toán 6.

Ta có bổ đề sau: Cho $F_n$ là dãy $Fibonacci$. Khi đó với mọi $n$ nguyên dương ta có:

$F_{1}^2+F_{2}^2+......+F_{n}^2=F_n.F_{n+1}$

Chứng minh bổ đề:

Ta chứng minh quy nạp theo $n$, dễ thấy đẳng thức đúng với $n=1$. Giả sử đẳng thức đã cho đúng tới $n=k, k\in N$, tức là:

$F_{1}^2+F_{2}^2+.......+F_{k}^2=F_k.F_{k+1}.$

Xét $n=k+1$ ta có:

$F_{1}^2+F_{2}^2+......+F_{k+1}^2$

$=(F_{1}^2+F_{2}^2+.........+F_{k}^2) + F_{k+1}^2$

$= F_k.F_{k+1} +F_{k+1}^2$

$=F_{k+1}(F_k+F_{k+1})$

$=F_{k+1}.F_{k+2}$.

Vậy đẳng thức đúng với $n=k+1$

Bổ đề được chứng minh.

Ta có một tiêu chuẩn bất khả quy của đa thức như sau:

Tiêu chuẩn Perron:

Cho đa thức nguyên $P(x)= \sum_{i=0}^{k} a_{i}x^{i} $ có $a_0 \ne 0$. Khi đó nếu

$|a_{n-1}| > |a_0|+|a_1|+....+|a_{n-2}| +|a_n|$

thì đa thức này bất khả quy.

Tiêu chuẩn này được chứng minh trong nhiều sách và tài liệu.

Quay trở lại bài toán

Theo tiêu chuẩn $Perron$ ta chỉ cần chứng minh

$F_{n+1}^2 > F_{1}^2+F_{2}^2+.......+F_{n}^2$

Mà theo bổ đề trên thì ta quy về $F_{n+1}^2 > F_n.F_{n+1}$ hay $F_{n+1} >F_{n}$ (luôn đúng với mọi $n$ nguyên dương)

Vậy bài toán được chứng minh.




#639749 VMF's Marathon Đa thức Olympic

Đã gửi bởi anhquannbk on 12-06-2016 - 09:05 trong Đa thức

$M$ là max các hệ số hay nghiệm thế bạn .

$M$ là max của các nghiệm, mình đã sửa lại.




#639738 VMF's Marathon Đa thức Olympic

Đã gửi bởi anhquannbk on 12-06-2016 - 08:45 trong Đa thức

Bài toán 7: (Sưu tầm) Cho $f(x)$ là đa thức nguyên có các nghiệm $ \alpha_1, \alpha_2,......, \alpha_n$.

Đặt $M=max|\alpha_i|$ với $ i \in \overline{1, n}.$

Chứng minh rằng nếu $f(x_0)$ là số nguyên tố với $x_0$ là số nguyên thỏa mãn $|x_0| > M+1$ thì $f(x)$ bất khả quy trên $\mathbb{Z}\left [ x \right ]$.




#643826 VMF's Marathon Hình học Olympic

Đã gửi bởi anhquannbk on 06-07-2016 - 10:42 trong Hình học

$ \boxed{\text{Lời giải bài toán 63}}$

Gọi $D$ là trung điểm $BC$, $E$ là điểm chính giữa cung $BC$ không chứa $A$ của $(O)$ , $F$ là trung điểm $MN$.

Gọi $I$ là tâm đường tròn nội tiếp tam giác $MNP$

$P$ là giao điểm của $BM$ và $CN$

Ta có $ \angle IMN =\dfrac{1}{2} \angle BMN= \dfrac{1}{2} (\angle BAM+ \angle ABM) =\dfrac{1}{2} \angle BAC$.

và $\angle HMN = 90^0 -\angle ONM =\dfrac{1}{2} \angle BAC$

Suy ra $\angle IMN =\angle HMN$. Tương tự ta được $I, H$ đối xứng nhau qua $AE$.

Do đó $F$ cũng là trung điểm $AE$.

Ta có $\dfrac{HO}{HF}= \dfrac{FO}{FH}-1=\dfrac{MF.cot \dfrac{A}{2}}{MF.tan\dfrac{A}{2}}-1 =\dfrac{2cos^2\dfrac{A}{2}-1}{1-cos^2\dfrac{A}{2}}$.

$\dfrac{DE}{DO}=\dfrac{R(1-cosA)}{RcosA} =\dfrac{2-2cos^2\dfrac{A}{2}}{2cos^2\dfrac{A}{2}-1}$

Suy ra $\dfrac{AF}{AE}. \dfrac{DE}{DO}.\dfrac{HO}{HF}=1$

Theo định lý $Menelaus$ cho tam giác $OME$ thì $A, H, D$ thẳng hàng.

hay $AH$ chia đôi $BC$.

Nếu phát biểu thêm gọi $S$ là giao điểm hai đường tiếp tuyến tại $B$ và $C$ của $(O)$ thì $AS$ là đường đối trng của tam giác $ABC$. Chứng minh $SA$ đi qua $I$.

và đây là bài toán chọn đội tuyển THPT chuyên sư phạm $2011-2012$ và olympic Đồng bằng Bắc Bộ năm nay.




#644084 VMF's Marathon Hình học Olympic

Đã gửi bởi anhquannbk on 08-07-2016 - 10:27 trong Hình học

Bài toán 71 là bài T12/459 trên báo THTT do thầy đề nghị thì phải.




#638616 VMF's Marathon Bất Đẳng Thức Olympic

Đã gửi bởi anhquannbk on 06-06-2016 - 23:04 trong Bất đẳng thức và cực trị

 

 Bài toán 27. (Sưu tầm) Chứng minh rằng với các số thực $x\geq 0$ tùy ý và số nguyên dương $n$ ta luôn có bất đẳng thức \[\left[nx\right]\geq \dfrac{\left[x\right]}{1}+\dfrac{\left[2x\right]}{2}+\dfrac{\left[3x\right]}{3}+\cdots +\dfrac{\left[nx\right]}{n}\]

Bài này là USAMO 1981 #5 đã được thảo luận tại

http://diendantoanho...n-nguyên/page-2

https://www.artofpro...h424778p2403926




#686335 USA TSTST 2017

Đã gửi bởi anhquannbk on 03-07-2017 - 13:13 trong Thi HSG Quốc gia và Quốc tế

Bài 1: Ta $ RC.RB=RE.RF $ nên $ R $ nằm trên trục đẳng phương của $ (ABC) $ đường tròn $ Euler $ của tam giác.

Dễ thấy $ \triangle APM \sim \triangle NPA $ nên $ PA^2=PM.PN $ tức $ P $ nằm trên trục đẳng phương của $ (ABC) $ đường tròn Euler

từ đó $ PR \perp OH $

Hình gửi kèm

  • 19692424_833326636815956_708596074_n.png



#640774 UEFA EURO 2016

Đã gửi bởi anhquannbk on 16-06-2016 - 22:49 trong Góc giao lưu

Cú sút phạt của Bale quá đẹp.




#639941 UEFA EURO 2016

Đã gửi bởi anhquannbk on 12-06-2016 - 23:17 trong Góc giao lưu

Đêm nay có ai thức xem cùng không :)




#634811 Tuần 4 tháng 5/2016: Tiếp nối câu chuyện về điểm và đường cố định

Đã gửi bởi anhquannbk on 22-05-2016 - 21:38 trong Chuyên mục Mỗi tuần một bài toán Hình học

Bài này làm em nhớ đến bài chứng minh $PP'$ đi qua một điểm cố định trong file điểm cố định của thầy Nguyễn Minh Hà.

Cách làm của em cũng dựa trên đó.

Có gì sai sót mong thầy và các bạn góp ý.

 

Ta thấy tồn tại một phép vị tự biến $\bigtriangleup ABC$ thành $\bigtriangleup DEF$.

Gọi phép vị tự đó tâm $O$ tỉ số $-k$

Khi đó ta có: $\vec{OD}=-k\vec{OA}$, $ \vec{OE}=-k\vec{OB}$, $ \vec{OF}=-k\vec{OC}$

Ta có $PA=QD$, $PB=QE$, $PC=QF$ nên

$(\vec{OA}-\vec{OP})^2=(\vec{OD}-\vec{OQ})^2$

$(\vec{OB}-\vec{OP})^2=(\vec{OE}-\vec{OQ})^2$

$(\vec{OC}-\vec{OP})^2=(\vec{OF}-\vec{OQ})^2$

Hay 

$\dfrac{1}{2}(OP^2-OQ^2+(1-k^2)OA^2)=\vec{OA}(k\vec{OQ}+\vec{OP})$

$\dfrac{1}{2}(OP^2-OQ^2+(1-k^2)OB^2)=\vec{OB}(k\vec{OQ}+\vec{OP})$              $(*)$

$\dfrac{1}{2}(OP^2-OQ^2+(1-k^2)OC^2)=\vec{OC}(k\vec{OQ}+\vec{OP})$

Bổ đề: Cho điểm $A$, vector $\vec{a}$ và số thực $k$. Khi đó tập hợp các điểm $M$ sao cho $\vec{AM}.\vec{a}=k $ là một đường thẳng.

Ta lấy các điểm $X, Y, Z$ sao cho $k\vec{OQ}+\vec{OP} =\vec{OX} +\vec{OP} =\vec{OY}$.

Gọi $Z$ là giao của $OY$ và $PQ$

Từ (*) ta suy ra

$\dfrac{1}{2}(1-k^2)(OA^2-OB^2)=(k\vec{OQ}+\vec{OP}).\vec{BA}= \vec{OY}. \vec{BA}.$

$\dfrac{1}{2}(1-k^2)(OA^2-OC^2)=(k\vec{OQ}+\vec{OP}).\vec{CA= \vec{OY}. \vec{CA}.}$      $(**)$

Theo bổ đề trên và theo $(**)$ thì $Y$ là giao của hai đường thẳng cố định.

Do đó $Y$ cố định.

 

 Kẻ $XT$ song song $PQ$ ta có $\bigtriangleup OZP= \bigtriangleup  YTX$

suy ra $OZ=YT$

tính được  $\dfrac{\overline{OZ}}{\overline{OY}}=\dfrac{1}{k+1}$

suy ra $Z$ cố định.

Vậy $PQ$ đi qua điểm cố định $Z$.

Hình gửi kèm

  • 13292893_611395079009114_266540931_n.png



#675475 Tuần 4 tháng 3/2017: $PQ$ luôn đi qua một điểm cố định khi $(K...

Đã gửi bởi anhquannbk on 27-03-2017 - 22:18 trong Chuyên mục Mỗi tuần một bài toán Hình học

Bài 1:

Trước hết ta  các bổ đề sau:

Bổ đề 1: Cho $ \triangle ABC$ nội tiếp đường tròn $ (O)$. Một đường tròn $ (K)$ bất kỳ qua $ B, C$ cắt $ CA, AB$ lần lượt tại $ E, F$, $ (AEF)$ cắt $ (O)$ tại $ G$ khác $A$. Khi đó ta  $ GEKB, GFKC$  các tứ giác nội tiếp.

Chứng minh:

Ta  $ \angle BGE+ \angle BKE= 2\angle BCA+ \angle BGA-\angle AGE= \angle BGE+\angle BCA =180^0$.

suy ra thứ giác $ GEKB$ nội tiếptương tự ta  $ GFKC$ nội tiếp.

Bổ đề 2: Cho $ \triangle ABC$ nội tiếp đường tròn $ (O)$. Một đường tròn $ (K)$ bất kỳ qua $ B, C$ cắt $ CA, AB$ lần lượt tại $ E, F$. Gọi $ H$  giao điểm $ BE$  $ CF$, $ I$  tâm đường tròn $ (KEF)$. Khi đó ta  $ H, I, O$ thẳng hàng.

Bổ đề này tham khảo tại https://artofproblemsolving.com/community/c6h454878p2556589

Bổ đề 3: Cho $ \triangle ABC$ cân tại $ A$ nội tiếp $(O)$.$ X$ bất , $ XB, XC$ cắt trung trục $ CA, CB$ tại $ N, M$. Lấy điểm $ Y$ sao cho $ BY \perp AM, CY \perp AN$. Khi đó ta  $ X, O, Y$ thẳng hàng.

Bổ đề này được suy ra từ bài toán của bạn Nguyễn Đức Bảo khi cho $ AB=AC$

Bài toán gốc https://artofproblemsolving.com/community/c6h1233260p6247264 

Quay trở lại bài toán.

Gọi $ M, N$ lần lượt  tâm đường tròn ngoại tiếp $ \triangle KCF, \triangle KBE$,$ Z $  giao điểm của $ EM$  $ FN$.

Ta  $ PM, PN$ lần lượt  trung trực của $KF, KE $. Áp dụng bổ đề 3 vào $ \triangle KEF$ cân ta được $ Z,H,O$ thẳng hàng,

từ đây theo bổ đề 2 ta  $ Z, Q, O$ thẳng hàng

Gọi $ S$  giao điểm của $ EF$  $BC$. Theo tính chất phương tích ta  $ AG, EF, BC$ đồng quy tại $ S$.

Từ đó $ SA.SG=SE.SF=SB.SC$

Xét phép nghịch đảo cực $ S$, phương tích trên biến $ (GBE)$ hay $ (KBE)$ thành $ (ACF)$ , biến $(GCF)$ hay $ (KFC)$ thành $ (ABE)$ Tức  biến tâm $ M, N$ của $ (KCF) , (KBE)$ thành tâm $ J,L $ của $ (ABE) , (AFC)$

$ f(M)f(N)--- f(J)f(L)$

Lại  $ EM \cap FN =Z , O, P$ thẳng hành nên $ FL \cap EJ=P , O, Q $ thẳng hàng.

Vậy $ PQ$ luôn đi qua tâm $ O$ cố định.




#688259 Trại hè phương nam

Đã gửi bởi anhquannbk on 21-07-2017 - 19:24 trong Thi HSG cấp Tỉnh, Thành phố. Olympic 30-4. Đề thi và kiểm tra đội tuyển các cấp.

Trại hè phương nam

 

Hình gửi kèm

  • 20228289_1788114334539306_1348208125173465749_n.jpg



#713461 TRẠI HÈ HÙNG VƯƠNG 2018

Đã gửi bởi anhquannbk on 29-07-2018 - 18:43 trong Thi HSG cấp Tỉnh, Thành phố. Olympic 30-4. Đề thi và kiểm tra đội tuyển các cấp.

TRẠI HÈ HÙNG VƯƠNG

LẦN THỨ XIV-PHÚ THỌ 2018

 

Câu 1(4 điểm). Giải hệ phương trình

$ \left\{\begin{matrix} &2x+y-297(x-y)\sqrt{x-y}=3 \\ &x(4x-2y+5)+2018\sqrt{x-y}=2y^2+5y \end{matrix}\right. $

Câu 2(4 điểm). Cho hàm số $f(x)=x^2+bx+c(b,c \in \mathbb{R})$.

Giả sử $\left\{ x \in \mathbb{R}, \vert f(x)\vert \le 1\right\}= \displaystyle \bigcup _{i=1}^n\left[ \alpha_i; \beta_i\right]\ne \emptyset$.

Chứng minh rằng $ \displaystyle \sum_{i=1}^n \vert \alpha_i - \beta_i \vert \le 2\sqrt{2} $
Câu 3(4 điểm). 
Cho tam giác $ABC$ nội tiếp đường tròn $(O)$, với $AB<AC<BC$. Gọi $D$  một điểm thuộc cạnh $ BC $, $ E $  một điểm trên tia $ BA $ sao cho $ BD=BE=AC $. $ P $  giao điểm của cạnh $ AC $ với $ (BDE) $. $ Q $  giao điểm thứ hai của $ BP $ với $ (O) $. Chứng minh rằng $ AQ+CQ=BP$.

Câu 4(4 điểm). Cho $p$  một số nguyên tố, $ n $  số nguyên dương lớn hơn $ 1 $  nguyên tố cùng nhau với $ p^{p+1}-p$. Gọi $\left\{a_1, a_2,...,a_k\right\} $  một hệ thặng  thu gọn theo modulo $ n $. Chứng minh rằng $ a_1^p+a_2^p+...+a_k^p $ chia hết cho $ n $.
Câu 5(4 điểm). Trong một buổi tập văn nghệ chào mừng chào mừng trại  Hùng Vương lần thứ $ XIV$,  $n(n \ge 3)$ học sinh được sắp thành một hàng ngangThầy Tuấn Anh chọn ra một đội văn nghệ gồm $k$ bạn từ $n$ bạn trên$ (1<k \le \dfrac{n+1}{2}) $ sao cho hai bạn đứng cạnh nhau thì không cùng được chọnHỏi thầy Tuấn  bao nhiêu cách chọn như vậy?




#649394 Trường hè toán học 2016 bài kiểm tra số 2

Đã gửi bởi anhquannbk on 13-08-2016 - 14:47 trong Thi HSG cấp Tỉnh, Thành phố. Olympic 30-4. Đề thi và kiểm tra đội tuyển các cấp.

Bài 5: Theo định lý $ Brocard $ ta có $ OG $ vuông góc với $ EF $ tại $ I $ đồng thời $ I $ cũng là điểm $ Miquel $ của tứ giác toàn phần $ ABCDEF$ suy ra các tứ giác $ BEIC, DCIF$ nội tiếp. Sử dụng hàng điểm ta chứng minh được $ IO$ là phân giác $ BID$ mà $ OB=OD$ nên tứ giác $ BIDO$ nội tiếp.

Gọi $ K$ là giao điểm của $ MN$ và $ IO$.

Ta chứng minh $ MINO$ nội tiếp.

Dễ thấy $ OM, ON$ là các trung trực của $ BC, CD$

Ta có: $ \angle IMO= \angle IMC +\angle OMC = 2\angle CEI+ \angle BIC  $

$ \angle INO= \angle INC +\angle CNO =2\angle IFC +\angle CID $

Suy ra $\angle OMI+ \angle INO = 2\angle CEI +2 \angle CFI +\angle BIC +\angle CID =2(180^0-\angle BCD) +\angle BID =\angle BOD +180^0- \angle BOD  =180^0 $

nên $ OMIN$ nội tiếp suy ra $ KI.KO= KM.KN$ dẫn đến $ K$ nằm trên trục đẳng phương của $ (OBID)$ và $ (CMN)$

mà $ PQ$ là trục đẳng phương của $ (CMN)$ và $ (OBID)$ suy ra $ PQ, IO, MN $ đồng quy.

Ta có $ \angle MON =\dfrac{1}{2} \angle BOD =\angle EAF$

$ \angle OMN =\dfrac{1}{2} \angle BMI =\angle BEI $

suy ra $ \bigtriangleup MON \sim \bigtriangleup EAF$.




#606697 TOPIC:CÁC CHUYÊN ĐỀ SỐ HỌC ÔN THI HSG TOÁN 9 VÀ VÀO LỚP 10

Đã gửi bởi anhquannbk on 02-01-2016 - 11:44 trong Số học

Mình xin góp một bài

Tìm tất cả các số tự nhiên $ n>1 $ sao cho $ (n-1)! $ chia hết cho $ n $




#657749 Topic: [LTDH] Mỗi ngày hai bất đẳng thức.

Đã gửi bởi anhquannbk on 13-10-2016 - 18:58 trong Bất đẳng thức và cực trị

Bài 93:

Đặt $ \sqrt{x}=a, \sqrt{y}=b, \sqrt{z}=c $

Khi đó ta $ a^2b^2+b^2c^2+c^2a^2=a^2b^2c^2 $

BĐT cần chứng minh trở thành

$ \sqrt{a^2+b^2c^2}+\sqrt{b^2+c^2a^2}+\sqrt{c^2+a^2b^2} \ge abc+a+b+c $

Áp dụng BĐT $ Mincopxki $ ta :

$ \sqrt{a^2+b^2c^2}+\sqrt{b^2+c^2a^2}+\sqrt{c^2+a^2b^2} \ge \sqrt{(a+b+c)^2+(ab+bc+ac)^2} $

$ (a+b+c)^2+(ab+bc+ac)^2 =(a+b+c)^2+a^2b^2+b^2c^2+c^2a^2 +2abc(a+b+c)=(a+b+c)^2 +a^2b^2c^2+2abc(a+b+c)=(abc+a+b+c)^2 $

$ \Longrightarrow \sqrt{a^2+b^2c^2}+\sqrt{b^2+c^2a^2}+\sqrt{c^2+a^2b^2} \ge abc+a+b+c $ (đpcm)

Dấu đẳng thức xảy ra khi $ x=y=z=3 $




#656462 Topic: [LTDH] Mỗi ngày hai bất đẳng thức.

Đã gửi bởi anhquannbk on 02-10-2016 - 20:17 trong Bất đẳng thức và cực trị

Bài 73:(Dùng pqr với xét hàm hơi cồng kềnh)

Đặt $ a+b+c=p=3, ab+bc+ac=q, abc=r $

Ta $ a^3+b^3+c^3 = p^3-3pq+3r =27-9q+3r $

Suy ra $ \sqrt[3]{\frac{a^3+b^3+c^3}{3}}+8\sqrt[3]{abc} = \sqrt[3]{9-3q+r} +8 \sqrt[3]{r} $ (*)

$ pq \ge 9r $ nên $ r \le \dfrac{q}{3} $ dẫn đến (*) $ \le \sqrt[3]{9-3q+\dfrac{q}{3}} + 8\sqrt[3]{\dfrac{q}{3}} $

Xét hàm $ f(q)= \sqrt[3]{9-\dfrac{8q}{3}} + 8\sqrt[3]{\dfrac{q}{3}} $ với $ q \in (0; 3] $

$ f'(q) = \dfrac{\dfrac{-8}{3}}{3\sqrt[3]{(9-\dfrac{8q}{3})^2}} + \dfrac{\dfrac{8}{3}}{3\sqrt[3]{(\dfrac{q}{3})^2}}$

$ f'(q)=0 \iff q=3 $

Do đó $ f(q) \le f(3) = 9 $

Vậy $ \sqrt[3]{\frac{a^3+b^3+c^3}{3}}+8\sqrt[3]{abc} \le 9 $, xảy ra khi $ a=b=c=1 $




#656752 Topic: [LTDH] Mỗi ngày hai bất đẳng thức.

Đã gửi bởi anhquannbk on 05-10-2016 - 12:41 trong Bất đẳng thức và cực trị

Bài 76:

Đặt $ x=\dfrac{a^2}{b^2+c^2}, y= \dfrac{b^2}{a^2+c^2}, z=\dfrac{c^2}{a^2+b^2}$

$ m=\dfrac{a}{b+c}, n= \dfrac{b}{a+c}, p=\dfrac{c}{a+b}$

Bất đẳng thức cần chứng minh: $ (x+\dfrac{1}{3})(y+\dfrac{1}{3})(z+\dfrac{1}{3}) \ge (m+\dfrac{1}{3})(n+\dfrac{1}{3})(p+\dfrac{1}{3}) $

$ \iff xyz +\dfrac{1}{3}(xy+yz+xz)+\dfrac{1}{9}(x+y+z) \ge mnp + \dfrac{1}{3}(mn+np+pm)+\dfrac{1}{9}(m+n+p)$

Ta một bất đẳng thức quen thuộc sau: $ \dfrac{a^2}{b^2+c^2} + \dfrac{b^2}{a^2+c^2} + \dfrac{c^2}{a^2+b^2} \ge \dfrac{a}{b+c} +\dfrac{b}{a+c} +\dfrac{c}{a+b} $, tức $ x+y+z \ge m+n+p $

Ta chứng minh $ xyz \ge mnp $, tức $ \dfrac{a^2b^2c^2}{(a^2+b^2)(b^2+c^2)(c^2+a^2)} \ge \dfrac{abc}{(a+b)(b+c)(c+a)} $

$ \iff (a^2+b^2)(b^2+c^2)(c^2+a^2) \ge abc(a+b)(b+c)(c+a) $

$ \iff  c^2(a-b)^2(a^2+ab+b^2) + b^2(a-c)^2(a^2+ac+c^2)+ a^2(b-c)^2(b^2+bc+c^2) \ge 0 $ (đúng)

Tương tự ta chứng minh được $ xy+yz+xz \ge mn+mp+np $

suy ra đpcm.




#656450 Topic: [LTDH] Mỗi ngày hai bất đẳng thức.

Đã gửi bởi anhquannbk on 02-10-2016 - 19:38 trong Bất đẳng thức và cực trị

Bài 72:

Áp dụng BĐT $ AM-GM $ ta

: $ 12\sqrt{xy}+ 16\sqrt{yz}=2.\sqrt{3x}.\sqrt{12y} \le 3x +12y$

$ 16\sqrt{yz}=2.2\sqrt{y}.4\sqrt{z} \le 4y +16z $

Suy ra $ 13x+12\sqrt{xy}+16\sqrt{yz} \le 16(x+y+z) $

$ \Longrightarrow P \ge \dfrac{24}{16(x+y+z)}-\dfrac{3}{\sqrt{x+y+z}} =\dfrac{3}{2(x+y+z)}-\dfrac{3}{\sqrt{x+y+z}} $

Đặt $ t=\sqrt{x+y+z} $

Xét hàm $ f(t)=\dfrac{3}{2t^2}-\dfrac{3}{t} $ với $ t \in (0; +\infty ) $

$ f'(t)= \dfrac{-3}{t^3}+\dfrac{3}{t^2} $

$ f'(t)=0 \iff t=1 $

Min $ P=\dfrac{-3}{2} $, xảy ra khi $ x=\dfrac{24}{29} , y= \dfrac{4}{29}, z= \dfrac{1}{29}$